Difference between revisions of "1959 AHSME Problems/Problem 7"

(Created page with "==Solution== If we let <math>a=3</math> and <math>d=1</math>, then we will get a <math>3</math>-<math>4</math>-<math>5</math> triangle, which is a right triangle. So, the answ...")
 
(Solution)
 
(2 intermediate revisions by the same user not shown)
Line 1: Line 1:
 
==Solution==
 
==Solution==
If we let <math>a=3</math> and <math>d=1</math>, then we will get a <math>3</math>-<math>4</math>-<math>5</math> triangle, which is a right triangle. So, the answer is <math>\boxed{\textbf{D}}</math>.
+
If we let <math>a=3</math> and <math>d=1</math>, then we will get a <math>3</math>-<math>4</math>-<math>5</math> triangle, which is a right triangle. So, the answer is <math>\boxed{\textbf{(D)} \ 3:1}</math>.

Latest revision as of 05:56, 24 September 2018

Solution

If we let $a=3$ and $d=1$, then we will get a $3$-$4$-$5$ triangle, which is a right triangle. So, the answer is $\boxed{\textbf{(D)} \ 3:1}$.